Proving (A-B) U C ≤ (A U B U C) - (A n B)

  • Thread starter Thread starter taylor81792
  • Start date Start date
Click For Summary
The discussion revolves around proving the set relationship (A - B) U C ⊆ (A U B U C) - (A ∩ B). Participants are attempting to clarify their understanding of set notation and the proof process. There's confusion regarding the correct interpretation of the problem, with some suggesting the use of cardinality, which is deemed irrelevant to the proof of set equality. The importance of correctly identifying set membership and relationships is emphasized, as well as the distinction between proving set inclusion versus equality. Overall, the conversation highlights the complexities involved in set theory proofs and the need for precise definitions and logical reasoning.
taylor81792
Messages
16
Reaction score
0

Homework Statement


The problem I have been given is to prove (A - B) U C is than less or equal to (A U B U C) - (A n B)

The Attempt at a Solution


I've tried starting off with just (A-B) U C. Then I would say how x ε c or x ε (A - B). Also if x ε a, then x ε c and x is not in b. If x ε c, since c is a subset of (A U B U C) , x ε (A U B U C). I don't know if this is right or where to go from here.
 
Physics news on Phys.org
taylor81792 said:

Homework Statement


The problem I have been given is to prove (A - B) U C is than less or equal to (A U B U C) - (A n B)
This is a statement about sets, so the relationship is \subseteq, not ≤.
taylor81792 said:

The Attempt at a Solution


I've tried starting off with just (A-B) U C. Then I would say how x ε c or x ε (A - B). Also if x ε a, then x ε c and x is not in b.
Try to be more careful with the names of the sets, which are A, B, and C, not a, b, and c.
taylor81792 said:
If x ε c, since c is a subset of (A U B U C) , x ε (A U B U C). I don't know if this is right or where to go from here.
 
i'd say that u can use:

AUBUC= lAl + lBl + lCl - lBnCl - lAnBl - lAnCl+lAnBnCl

but I'm not 100% positive just trying to give some help :)
 
My professor said we can also try to prove or find a counterexample to this statement. Let A, B and C be sets. Then (A-B) U C = (A U B U C) - (A n B). I'm not really sure what she means by counterexample.
 
mtayab1994 said:
i'd say that u can use:

AUBUC= lAl + lBl + lCl - lBnCl - lAnBl - lAnCl+lAnBnCl

but I'm not 100% positive just trying to give some help :)
Not only does that not help, it makes no sense. The left side is a set, the right side is a number.

Even if you meant |AUBUC| that is irrelevant to the problem. Showing that two sets have the same size does not prove they are the same set.
 
Question: A clock's minute hand has length 4 and its hour hand has length 3. What is the distance between the tips at the moment when it is increasing most rapidly?(Putnam Exam Question) Answer: Making assumption that both the hands moves at constant angular velocities, the answer is ## \sqrt{7} .## But don't you think this assumption is somewhat doubtful and wrong?

Similar threads

Replies
7
Views
1K
  • · Replies 8 ·
Replies
8
Views
2K
  • · Replies 9 ·
Replies
9
Views
2K
  • · Replies 4 ·
Replies
4
Views
3K
  • · Replies 2 ·
Replies
2
Views
3K
  • · Replies 7 ·
Replies
7
Views
1K
Replies
3
Views
2K
  • · Replies 16 ·
Replies
16
Views
5K
  • · Replies 11 ·
Replies
11
Views
2K
  • · Replies 1 ·
Replies
1
Views
2K